Q12

 
clarafok
Thanks Received: 5
Forum Guests
 
Posts: 98
Joined: December 27th, 2010
 
 
trophy
Most Thankful
trophy
First Responder
 

PT28, P2, Q12

by clarafok Wed Feb 02, 2011 1:22 pm

hello,

i'm really confused as to why the answer is C. doesn't C mean that the passage needs to have info on how major volcanic eruptions do have different effects on regional temperature? or does it just need to make no mention of discernible effects?

and i kind of get why E is wrong but at the same time i don't really. is it because line 32-37 says that major explosions caused only a small drop in avg. temperature that i was supposed to infer that minor eruptions have no discernible effect in the opposite hemisphere?

thanks in advance!
 
giladedelman
Thanks Received: 833
LSAT Geek
 
Posts: 619
Joined: April 04th, 2010
 
This post thanked 1 time.
 
 

Re: PT28, P2, Q12

by giladedelman Mon Feb 07, 2011 7:37 pm

Thanks for your question!

Remember, here we're looking for the claim for which the passage offers the LEAST support.

(C) is correct because the passage does not support the claim that major eruptions have no discernible effect on regional temperature -- it says the exact opposite! Even though the effect is smaller than previously thought, we still know that major eruptions cause a drop in temperature. So there is a discernible effect. (Discernible doesn't mean "different," it means we're able to point it out.)

(E) is incorrect because the passage explicitly states that "minor eruptions have no discernible effect on temperature." So forget about the hemisphere part; they have no effect, period.

Does that answer your question?
 
evelina.chang
Thanks Received: 1
Forum Guests
 
Posts: 12
Joined: November 19th, 2011
 
 
 

Re: Q12

by evelina.chang Fri Jun 01, 2012 3:54 am

For this one, I chose (A) because I took the main idea of the passage and applied it to this question.

However, upon further review, is (A) wrong because the information in the passage does provide support for it in Paragraph 1 (that the author later tries to disprove)?

Thanks!
 
aerialstrong
Thanks Received: 0
Forum Guests
 
Posts: 11
Joined: August 26th, 2012
 
 
 

Re: Q12

by aerialstrong Sat Aug 31, 2013 2:55 pm

evelina.chang Wrote:For this one, I chose (A) because I took the main idea of the passage and applied it to this question.

However, upon further review, is (A) wrong because the information in the passage does provide support for it in Paragraph 1 (that the author later tries to disprove)?

Thanks!


A is wrong because between line 30-37: Minor eruptions have no discernible effect; but major one does- smaller than expected but discernible.
 
ReginaP412
Thanks Received: 0
Vinny Gambini
Vinny Gambini
 
Posts: 15
Joined: June 23rd, 2020
 
 
 

Re: Q12

by ReginaP412 Fri Dec 25, 2020 1:12 pm

Hi, could someone explain why the part of the passage that says a major explosion causes 'a smaller than expected in the average temperature in the *hemisphere (northern or southern) of the eruption*" -- doesn't eliminate A that says the discernible effect is on 'global temperatures'.

(I understand why C is correct... just not how to eliminate A)

thanks!
 
Misti Duvall
Thanks Received: 13
Atticus Finch
Atticus Finch
 
Posts: 191
Joined: June 23rd, 2016
 
 
 

Re: Q12

by Misti Duvall Sun Jan 10, 2021 7:32 pm

ReginaP412 Wrote:Hi, could someone explain why the part of the passage that says a major explosion causes 'a smaller than expected in the average temperature in the *hemisphere (northern or southern) of the eruption*" -- doesn't eliminate A that says the discernible effect is on 'global temperatures'.

(I understand why C is correct... just not how to eliminate A)

thanks!


Sure! I believe the text you quoted is a fair way to eliminate answer choice A. If it causes a smaller temperature drop than expected, that's still a discernible effect, so A has some support and can be eliminated.
LSAT Instructor | Manhattan Prep